2006 AMC 12A Problems/Problem 25

Revision as of 21:06, 17 February 2012 by Baijiangchen (talk | contribs) (Solution)

Problem

How many non- empty subsets $S$ of $\{1,2,3,\ldots ,15\}$ have the following two properties?

$(1)$ No two consecutive integers belong to $S$.

$(2)$ If $S$ contains $k$ elements, then $S$ contains no number less than $k$.

$\mathrm{(A) \ } 277\qquad \mathrm{(B) \ } 311\qquad \mathrm{(C) \ } 376\qquad \mathrm{(D) \ } 377\qquad \mathrm{(E) \ }  405$

Solution

This question can be solved fairly directly by casework and pattern-finding. We give a somewhat more general attack, based on the solution to the following problem:

How many ways are there to choose $k$ elements from an ordered $n$ element set without choosing two consecutive members?

Note that if $n < 2k - 1$, the answer will be 0. Otherwise, the $k$ elements we choose define $k + 1$ boxes into which we can drop the $n - k$ remaining elements, with the caveat that each of the middle $k - 1$ boxes must have at least one element. This is equivalent to dropping $n - 2k + 1$ elements into $k + 1$ boxes, where each box is allowed to be empty. And this is equivalent to arranging $n - k + 1$ objects, $k$ of which are dividers, which we can do in $F(n, k) = {n - k + 1 \choose k}$ ways.

Now, looking at our original question, we see that the thing we want to calculate is just $F(15, 1) + F(14, 2) + F(13, 3) + F(12, 4) + F(11, 5) = {15\choose 1} + {13\choose2} + {11\choose 3} + {9\choose 4} + {7 \choose 5} = 15 + 78 + 165 + 126 + 21 = 405 \Longrightarrow \mathrm{(E)}$


Another way to think about it that you want to choose $k$ numbers out of $n$ with no consecutive numbers. Each given each configuration, subtract $i-1$ from the $i$-th element in your subset. This converts your configuration into a configuration with $k$ elements where the largest possible element is $n-k+1$, with no restriction on consectutive numbers. Since this process is easily reversible, we have a bijection. Without consideration of the second condition, we have: ${15 \choose 1} + {14 \choose 2} + {13 \choose 3} + ... + {9 \choose 7} + {8 \choose 8}$

Now we examine the second condition. It simply states that no element in our original configuration (and hence also the modified configuration, since we don't move the smallest element) can be less than $k$, which translates to subtracting $k$ from the "top" of each binomial coefficient. Now we have, after we cancel all the terms ${n \choose k}$ where $n < k$, ${15 \choose 1} + {13 \choose 2} + {11 \choose 3} + {9 \choose 4} + {7 \choose 5}=  15 + 78 + 165 + 126 + 21 = \boxed{405} \Longrightarrow \mathrm{(E)}$

See also

2006 AMC 12A (ProblemsAnswer KeyResources)
Preceded by
Problem 24
Followed by
Last Question
1 2 3 4 5 6 7 8 9 10 11 12 13 14 15 16 17 18 19 20 21 22 23 24 25
All AMC 12 Problems and Solutions